Which one of the following can be inferred regarding the two proposals discussed in the passage?

Eugene on August 14 at 11:27AM

Why not B?

I feel like B is directly supported by the passage. Can someone please explain why B is wrong?

Reply
Create a free account to read and take part in forum discussions.

Already have an account? log in

Emil-Kunkin on August 14 at 07:34PM

What support do you see for B? I think it's a great exercise to try to actually articulate why you like an incorrect answer choice, because for a must be true the burden of proof is on the answer choice to prove itself correct, not for you to prove it wrong.